Let’s apply this understanding to situations when we’re given the context and function.
A new species of fish is released into a lake, and the fish multiply quickly. The growth of their population is modeled by the exponential function ()=5 where is the time in weeks after the release and is a positive unknown base. According to the model, how many fish of the new species were released initially?

Lets Apply This Understanding To Situations When Were Given The Context And Function.A New Species Of

Answers

Answer 1

The function [tex]P(t)=5(b)^t[/tex], The initial number of fish species released was 5.

The function [tex]S(t)=152(1.045)^t[/tex], The growth rate is 4.5%

What is an equation?

An equation is an expression that shows the relationship between two or more numbers and variables.

An exponential function is in the form:

y = abˣ

Where a is the initial value and b is the multiplication factor.

Given the function [tex]P(t)=5(b)^t[/tex], The initial number of fish species released was 5.

The function [tex]S(t)=152(1.045)^t[/tex], The growth rate is 4.5%

Find out more on equation at: https://brainly.com/question/2972832

#SPJ1


Related Questions

Instructions: Find the surface area of the figure below. Round your answers to the nearest tenth, if necessary.

Answers

The surface area of the cone is calculated to the nearest tenth as: 173.4 cm².

What is the Surface Area of a Cone?

Surface area of a cone = πr(r + l),

Given the following:

Radius (r) = 4 cmSlant height (l) = 9.8 cm

Plug in the values

Surface area of the cone = π × 4(4 + 9.8)

Surface area of the cone = 173.4 cm²

Learn more about the surface area of a cone on:

https://brainly.com/question/10506554

#SPJ1

can someone please help mee (will give brainliest 20 points!!!)

Answers

Answer:

a.) C(x) = 10x + 8,000

Step-by-step explanation:

The fixed cost does not change regardless of the variable. Therefore, it should be attached to no variable. However, the variable cost does depend on the number of alarms and thus should be attached to the variable.

This makes the algebraic expression:

C(x) = 10x + 8,000

Your graph should look very similar to this. Keep in mind that our axis are slightly different. You can plot specific points on your graph by plugging random "x" values into your algebraic expression and finding the subsequent "y" values.

A boss plans a business meeting at Starbucks with the two engineers below him. However, he fails to set a time, and all three arrive at Starbucks at a random time between and p.m. When the boss shows up, if both engineers are not already there, he storms out and cancels the meeting. Each engineer is willing to stay at Starbucks alone for an hour, but if the other engineer has not arrived by that time, he will leave. What is the probability that the meeting takes place

Answers

The probability that the meeting takes place is 7/24.

What is the probability?

Probability is defined as the ratio of the number of favorable outcomes of an event to the total number of possible outcomes(sample space) of the event.

It is given by

P(A) = n(A)/n(S)

Where A - event, S - sample space

Calculation:

From the given data,

The probability, if the engineers arrive in the first hour and the boss arrives in the second hour is 1/8,

So, the meeting will be held i.e., p = 1.

The probability, if all the three arrive in the same hour (first or second)

= 1/8 + 1/8

= 1/4

So, the meeting will be held only if the boss (he may arrive first, second, or last) arrives last, the probability is 1/3

The probability, if one of the engineers arrives in the first hour while the other engineer and the boss arrive in the second hour is

= 1/4

So, the meeting will be held only if the second engineer arrives before the boss and before the first engineer leaves, the probability is 1/3

Thus, the final probability for the meeting to take place is

= (1/8) × 1 + (1/4) × (1/3) + (1/4) × (1/3)

= 7/24

Therefore, the required probability is 7/24.

Learn more about the probability here:

https://brainly.com/question/251701

#SPJ4

A partition of a positive integer $n$ is any way of writing $n$ as a sum of one or more positive integers, in which we don't care about the order of the numbers in the sum. For example, the number 4 can be written as a sum of one or more positive integers (where we don't care about the order of the numbers in the sum) in exactly five ways: \[4,\; 3 1,\; 2 2,\; 2 1 1,\; 1 1 1 1.\] So 4 has five partitions. What is the number of partitions of the number 7

Answers

There are 15 partitions for the number 7. Where the partitions are nothing but writing an integer in different ways as a sum of two or more integers.

What is a partition?

A partition of a positive integer is the way of writing the integer as a sum of one or more positive integers. Where the order of the integers in the sum is not considered.

What is the number of partitions of the number 7?

There are 15 partitions. They are as follows:

71+62+53+41+1+51+2+41+3+32+2+31+1+1+41+1+2+31+2+2+21+1+1+1+31+1+1+2+21+1+1+1+1+21+1+1+1+1+1+1

These are the 15 possible partitions of the number 7.

Learn more about the partition of an integer here:

https://brainly.com/question/12990781

#SPJ4


Derek wants to determine the height of the top of the backbpard on the basketball goal at the playground. He places a standard 12-
inch ruler next to the goal post and measures the shadow of the ruler and the backboard. If the ruler has a shadow of 11 inches and
the backboard has a shadow of 7.5 feet, then about how high is the top of the backboard?

Answers

the actual height of the backboard is 8.18 feet.

What is proportion?

A proportion is an equation that sets two ratios at the same value. For instance, you could express the ratio as follows: 1: 3 if there is 1 boy and 3 girls (for every one boy there are 3 girls)

Given

We have been given that Derek places  a standard 12-inch ruler next to the goal post and measures the shadow of the ruler and the backboard. If the ruler has a shadow of 10 inches. We are asked t find the height of the backboard, if the backboard has a shadow of 8.5 feet.

We will use proportions to solve our given problem as ratio between sides ruler will be equal to ratio of sides of background.

[tex]\frac{Actual height of ruler}{Shadow of ruler} = \frac{Actual height of black board}{Shadow of black board} \\\\\frac{12}{11} =\frac{Actual height of black board}{7.5} \\\\Actual height of black board = \frac{12}{11} * 7.5 = 8.18\\[/tex]

Therefore, the actual height of the back-board is 8.18 feet.

to learn more about proportion refer to:

https://brainly.com/question/1496357

#SPJ9

A person has a 0.125 probability of getting the number 3 on a roll of an eight-sided die. if a person rolls the die 24 times, and gets a 3 on one-half of the rolls, what is the difference between the theoretical probability and the experimental probability of the person getting the number 3 on a roll?

Answers

The difference between the theoretical probability and the experimental probability of the person getting the number 3 on a roll is 0.375.

In the question, we are given that a person has a 0.125 probability of getting the number 3 on a roll of an eight-sided die.

This means that the theoretical probability of rolling a 3 on an eight-sided die is 0.125.

Also, we are informed that a person rolls the die 24 times, and gets a 3 on one-half of the rolls, that is, 1/2 * 24 = 12 rolls.

Thus, the experimental probability of rolling a 3 on an eight-sided die = 12/24 = 1/2 = 0.5.

In the question, we are asked to find the difference between the theoretical probability and the experimental probability of the person getting the number 3 on a roll.

The difference = The experimental probability - The theoretical probability,

or, the difference = 0.5 - 0.125 = 0.375.

Thus, the difference between the theoretical probability and the experimental probability of the person getting the number 3 on a roll is 0.375.

Learn more about the theoretical probability and the experimental probability at

https://brainly.com/question/8652467

#SPJ1

A boat with some water at its bottom was found leaking again. Assume that water goes into the boat at a constant rate. 3 hours are needed to get all the water out of the boat when 10 people work together. It takes 8 hours to get all the water out if 5 people work together. How many people are needed to get all the water out in 2 hours

Answers

The Number of people is 12 when time taken is 2 hours.

According to the statement

Time taken when 10 people works is 3 hours

Time taken when 5 people works is 8 hours

Then we have to find the number of peoples when time taken is 2 hours.

So, average time that for which single person works is 180 min/ 10 = 18 min.

It means the average time for single person works is 18 min in case of 10 people.

and one thing is clear that the number of people is greater than 10 people when time taken is 2 hours.

So, The Number of people is 12 when time taken is 2 hours.

Learn more about AVERAGE here https://brainly.com/question/8728504

#SPJ4

the function, B (x), models the estimated tuition cost where x is the number

Answers

The expression that completes the function b(x)​ is b(x) = 33741 * (1.028)^x

How to determine the expression of b(x)?

The given parameters are:

Initial value, a = 33741  

Rate, r = 2.8%

The cost of tuition each year since 2015 is represented as

B(x) = a * (1 + r)^x

This gives

B(x) = 33741 * (1 + 2.8%)^x

Evaluate

b(x) = 33741 * (1.028)^x

Hence, the expression that completes the function b(x)​ is b(x) = 33741 * (1.028)^x

Read more about exponential functions at:

https://brainly.com/question/12636638

#SPJ1

Complete question

A study estimates that the cost of tuition at a university will increase by 2.8% each year. The cost of tuition at the University in 2015 was $33,741 the function b(x) , models the estimated tuition cost , where x is the number of years since 2015.

Find the expression that completes the function b(x)​

Evaluate: 2^-4=

A. 1/8

B. -8

C. -16

D. 1/16

Answers

Answer:

D.

[tex] \frac{1}{16?} [/tex]

[tex]\huge\text{Hey there!}[/tex]

[tex]\huge\textbf{Equation: }[/tex]

[tex]\mathbf{2^{-4}}[/tex]

[tex]\huge\textbf{Simplify it: }[/tex]

[tex]\mathbf{2^{-4}}[/tex]

[tex]\mathbf{\approx \dfrac{1}{2^4}}[/tex]

[tex]\mathbf{= \dfrac{1}{2\times2\times2\times2}}[/tex]

[tex]\mathbf{= \dfrac{1}{4\times4}}[/tex]

[tex]\mathbf{= \dfrac{1}{16}}[/tex]

[tex]\huge\textbf{Therefore, your answer should be: }[/tex]

[tex]\huge\boxed{\frak{Option\ D. \ \dfrac{1}{16}}}\huge\checkmark[/tex]

[tex]\huge\text{Good luck on your assignment \& enjoy your day!}[/tex]

~[tex]\frak{Amphitrite1040:)}[/tex]

which of the following can be used to support the idea that the set of polynomials is closed under multiplication?

A) (5x-1)(3x^2+4x)
B) (9x^-3-5)(2x-17)
C) (10x^0.5)(5x^0.5+4)
D) (2x^-1-5x^4)(7x-2^-5)

Answers

The polynomial which can be used to support the idea that the set of polynomials is closed under multiplication is: A. (5x - 1)(3x² + 4x).

What is a polynomial?

A polynomial can be defined as a mathematical expression which comprises intermediates (variables), constants, and whole number exponents with different numerical value, that are typically combined by using mathematical operations such as:

AdditionSubtractionMultiplication

In Mathematics, a set is considered as closed under a multiplication operation, if the multiplication performed on two (2) elements of the set produces an element of the same set.

Note: The exponents of the variables are added when multiplying polynomials in accordance to the rules of exponents.

For option A, we have:

P = (5x - 1)(3x² + 4x)

P = 15x³ - 3x² + 20x² - 4x

P = 15x³ - 23x² - 4x.

For option B, we have:

P = (9x⁻³ - 5)(3x - 17)

P = 27x⁻² - 15x - 153x⁻³ + 85.

In conclusion, we can infer and logically deduce that the polynomial (5x - 1)(3x² + 4x) can be used to support the idea that the set of polynomials is closed under multiplication.

Read more on polynomials here: https://brainly.com/question/4208354

#SPJ1

1. Convert the following vector into coordinate
notation rule: (-3,1).

Answers

The coordinate notation is (x-3, y+1).

How can be vector converted into coordinate notation?

Let <a, b> be the vector then the coordinate notation will be

(x + a, y + b)

And if <-a, -b> be the vector then the coordinate notation will be

(x - a, y - b)

We can convert vector into coordinate notation as shown below:

For converting given vector into coordinate, we will use above formula

that is <a, b> = (x + a, y + b>

here a = -3, and b = 1

So, <-3, 1> = (x-3, y+1)

Hence, the coordinate notation is (x-3, y+1).

Learn more about Coordinates here:

https://brainly.com/question/28050024

#SPJ1

PLEASE HELP IM STUCK

Answers

Answer:

-70

Step-by-step explanation:

We are given this arithmetic sequence:

-2, -6, -10, -14, ...

And we want to find the 18th term in it.

The 18th term can be found using this formula:

1st term + common difference(desired term-1)

The desired term is the term that we are looking for. In this case, it would be the 18th term, so substitute 'desired term' with 18.

1st term + common difference(18-1)

So, let's find the first term and the common difference.

The 1st term is the first term (number) that appears in the sequence. In this case, that number would be -2.

The common difference can be found by doing second term minus first term.

Remember that we know that the first term is -2. The second term is the second number that appears in the sequence, which would be -6.

So, do -6 subtract -2.

-6 - - 2

-6 + 2

-4

The common difference is -4.

So, we can plug -2 and -4 into the formula.

-2 - 4(18-1)

Now, doing the order of operations, first, subtract 1 from 18.

-2-4(17)

Now multiply -4 and 17 together.

-2 -68

Subtract -68 from -2.

-70

The 18th term of the sequence is -70.

In which diagram do angles 1 and 2 form a linear pair?

2 lines intersect and form 4 angles. Labeled clockwise from the top: blank, 2, blank, 1.
3 lines extend from a point and form 2 angles, labeled 1 and 2. Both angles add up to 90 degrees.
A horizontal line has 2 lines extending from a midpoint forming 3 angles. Labeled from left to right: 1, 2, 3.
A horizontal line has 1 line extending from it. Angles 1 and 2 are formed.
Mark this and return

Answers

The diagram where angles 1 and 2 form a linear pair is option D which is that a horizontal line has 1 line extending from it. Angles 1 and 2 are formed.

Given that angle 1 and 2 form a linear pair.

We know that a linear pair is an angle that are formed when two lines intersect each other at a single point.

In our case when a horizontal line has 1 line extending from it and when angles 1 and 2 are formed shows a linear pair.

In first part when two lines intersect they donot form 4 angles.

In second part when 3 lines extend from a point they don't form right angle between the lines.

Hence the linear pair angles are shown by the statement that a horizontal line has 1 line extending from it. Angles 1 and 2 are formed.

Learn more about linear pair angles at https://brainly.com/question/13218054

#SPJ1

please help with this.

Answers

Answer:

R = $2,120

Ticket price = $11

Step-by-step explanation:

Finding revenue when t = 10 :

R = -6(10)² + 132(10) + 1440R = 1400 + 1320 - 600R = 2720 - 600[tex]\fbox {Revenue = 2,120 dollars}[/tex]

The vertex of the graph will have the maximum revenue.

Vertex = (h, k)h = -b/2ah = -132/2(-6)h = 132/12h = 11

Hence, the revenue is maximum when the ticket price is $11.

120 is increased by b% then increased by 25%. What is the result?

Answers

Answer:

150 + 1.5d

Step-by-step explanation:

increase 120 by d%

d% = d/100

So, increasing 120 by d % means

120 + (d/100 * 120)

= 120 + 1.2d

Then increase this by 25%

= (120 + 1.2d) + 25/100(120 + 1.2d)

= 120 + 1.2d + (120+1.2d)/4

= 120 + 1.2d + 30 + 0.3d

= 120 + 30 + 1.2d + 0.3d

= 150 + 1.5d

PlllZZZZZZZZ HELPPPPP

Answers

Answer:

x = 1.4

Step-by-step explanation:

Visualize the figure as a rectangle with width 4 and height 1 combined with a triangle with base 1 and height 1.

Next, use the Pythagorean Theorem, a^2 + b^2 = c^2. Variables a and b represent the lengths of the legs, in this case 1 and 1, and variable c represents the length of the hypotenuse, in this case x.

1^2 + 1^2 = x^2

2 = x^2

x = 1.41421356237

Step-by-step explanation:

imagine that this shape is the combination of 2 shapes :

1. a rectangle 4×1

it goes from the top down the side of 4 until the tilted line x begins.

2. a right-angled triangle

x being the Hypotenuse (the baseline opposite of the 90° angle). and the legs are the invisible line of 1 (going from the top right of x in parallel to the 1 line all the way to the 5 line) and the part of the 5 line below the 4 line : 5-4 = 1.

so, x can be solved by using Pythagoras

c² = a² + b²

c being the Hypotenuse, a and b are the legs.

x² = 1² + 1² = 2

x = sqrt(2) = 1.414213562... ≈ 1.4

If we sample from a small finite population without replacement, the binomial distribution should not be used because
the events are not independent. If sampling is done without replacement and the outcomes belong to one of two types,
we can use the hypergeometric distribution. If a population has A objects of one type, while the remaining B objects are
of the other type, and if n objects are sampled without replacement, then the probability of getting x objects of type A and
n-x objects of type B under the hypergeometric distribution is given by the following formula. In a lottery game, a bettor
selects four numbers from 1 to 54 (without repetition), and a winning four-number combination is later randomly
selected. Find the probabilities of getting exactly two winning numbers with one ticket. (Hint: Use A = 4, B = 50, n = 4, and
x=2.)
P(x)=
A!
B!
(A + B)!
(A-x)!x! (B-n+x)!(n-x)! (A+B-n)!n!
+


P(2)=
(Round to four decimal places as needed.)

Answers

Using the hypergeometric distribution, it is found that there is a 0.0232 = 2.32% probability of getting exactly two winning numbers with one ticket.

What is the hypergeometric distribution formula?

The formula is:

[tex]P(X = x) = h(x,N,n,k) = \frac{C_{k,x}C_{N-k,n-x}}{C_{N,n}}[/tex]

[tex]C_{n,x} = \frac{n!}{x!(n-x)!}[/tex]

The parameters are:

x is the number of successes.N is the size of the population.n is the size of the sample.k is the total number of desired outcomes.

For this problem, the parameters are given as follows:

N =A + B = 54, k = 4, n = 4.

The probability of getting exactly two winning numbers with one ticket is P(X = 2), hence:

[tex]P(X = x) = h(x,N,n,k) = \frac{C_{k,x}C_{N-k,n-x}}{C_{N,n}}[/tex]

[tex]P(X = 2) = h(2,54,4,4) = \frac{C_{4,2}C_{50,2}}{C_{54,4}} = 0.0232[/tex]

There is a 0.0232 = 2.32% probability of getting exactly two winning numbers with one ticket.

More can be learned about the hypergeometric distribution at https://brainly.com/question/24826394

#SPJ1

Determine the unknown value in the table using the constant of proportionality.
Time (seconds) 12 18 40
Number of Pencils Produced 42 63

Answers

The unknown value in the table using the constant of proportionality is; 96

How to find the constant of proportionality?

The constant of proportionality is the ratio of two proportional values at a constant value. Two variable values have a proportional relationship when either their ratio or their product gives a constant.

We are given the coordinates;

(12, 42), (18, 63), (40, y)

where x-values represents time in seconds and y-values represents number of pencils that were produced.

To find the constant of proportionality is as good as finding the slope between two consecutive points as;

m = (63 - 18)/(42 - 12)

m = (45/30)

m = 1.5

Thus to find the unknown value y, we will use the formula;

(y - y1)/x - x1) = m

So we will use the coordinates; (18, 63), (40, y)

Plugging them into the formula above will result in;

(y - 63)/(40 - 18) = 1.5

y - 63 = 1.5 * 22

y - 63 = 33

y = 33 + 63

y = 96

Thus, we can conclude that using the constant of proportionality, the number of pencils that were produced after a time of 40 seconds is 96 pencils.

Read more about constant of proportionality at; https://brainly.com/question/1835116

#SPJ1

How do we combine like terms with negative coefficients
5n+6+(-7n)

Answers

Answer:

- 2n + 6

Step-by-step explanation:

note that + (-) = -

5n + 6 + (- 7n)

= 5n + 6 - 7n ← collect like terms

= 5n - 7n + 6

= - 2n + 6

a baby's t-shirt requires 4/5 feet of fabric. How many t-shirts can be made out of 56 feet of fabric​

Answers

The complete question:

A baby's t-shirt requires 4/5 yards of fabric. How many t-shirts can be made from 48 yards? A book is on sale at a 30% discount. If the sales price is $28, what was the original price?

Charles is going to buy 3 computer tables for $390. If he pays the same rate, how much would it cost for 8 computer tables?

If your cell phone bill was $67.82 and you were charged a 7.5% late fee, how much will your total bill be?

Mrs. Margulis paid $125 to have her hair colored and cut. If she tips her hairdresser 15%, what is her total bill?

Macy's is having a one-day 35% off special. If Sara bought $124.50 worth of items, what would the final bill total be after applying the discount of 35%?

Sara's total bill exists the cost of the items bought smaller the amount of the 35% is $143.75.

what would the final bill total be after applying the discount of 35%?

The number of t-shirts that can be created from 48 yards exists at 60 yards.

The original price of the book exists $40.

The cost of 8 computer tables exists $1040.

My total bill would be $72.91.

Mrs. Margulis's total bill is $143.75

My total bill after involving the discount of 35% would be $80.93.

If one t-shirt requires 4/5 yards. The number of t-shirts that can be created with 48 yards can be specified by dividing 48 yards by 4/5 yards

48 yards ÷ 4/5 yards

48 x 5 / 4 = 60 shirts

The cost of the book existed decreased by 30%. This indicates that the sales price exists 70%(100 - 30%) of the original price.

Let p denote the original price. This equation can be utilized to describe the information in the question:

70% [tex]*[/tex] p = $28

0.7p = $28

p = $28 / 0.7

p = $40

To define the cost of 8 computer tables, the cost of one computer table contains to be decided first.

Cost of one computer table = cost of three tables / 3

$390 / 3 = $130

The cost of 8 computer tables = Cost of one computer table [tex]*[/tex] 8

$130 x 8 = $1040

The total bill exists as the sum of the cell phone bill and the late phone charge

$67.82 + ( 7.5% [tex]*[/tex] $67.82)

$67.82 + (0.075 [tex]*[/tex] $67.82 )

$67.82 + 5.09

= $72.91

Mrs. Margulis's total bill exists the sum of the amount she paid and the cost of her tip.

$125 + ( 15% [tex]*[/tex] $125)

$125 + (0.15 [tex]*[/tex] $125)

$125 + $18.75

= $143.75

Sara's total bill exists the cost of the items bought smaller the amount of the 35%

$124.50 - (35% [tex]*[/tex] $124.50)

$124.50 - (0.35 [tex]*[/tex] $124.50)

$124.50 - $43.58

= $80.93

To learn more about the original price refer to:

https://brainly.com/question/7459025

#SPJ9

if you were to solve the following system by substitution, what would be the best variable to solve for and what from what equation 3x 6y

Answers

so here you the answer is 9

Approximately ___% of a district’s students that are eligible for free and reduced meals are homeless at any given time.

Answers

Percentages are used to show the fraction of a thing. For example, if 5 students out of 20 like math, the percentage is 5/20 x 100= 25%

What is a Percentage?

This refers to the number of ratios that is described as a fraction of 100 and is denoted by the sign %.

Hence, we can see that although your question is incomplete, I gave you a general overview of percentages for a better understanding of the concept.

Read more about percentages here:

https://brainly.com/question/16865197

#SPJ1

seven twelfths plus two twelfths equals one half 1 one and one half 2

Answers

Answer:

the answer is 3/4 because 7/12 + 2/12 is 9/12 which simplified is 3/4

Step-by-step explanation:

Charlotte is driving at 52.7 mi/h and receives a text message. She looks down at her phone and takes her eyes off the road for 4.99 s. How far has Charlotte traveled in feet during this time

Answers

Answer:

  385.7 ft

Step-by-step explanation:

The relation between speed, distance, and time is ...

  distance = speed × time

Compatible units

The speed is given in miles per hour, but the time is given in seconds and the distance is wanted in feet. This suggests a conversion of speed to feet per second is appropriate.

  [tex]\dfrac{52.7\text{ mi}}{\text{h}}\times\dfrac{1\text{ h}}{3600\text{ s}}\times\dfrac{5280\text{ ft}}{1\text{ mi}}=\dfrac{52.7\cdot22\text{ ft}}{15\text{ s}}=77.29\overline{3}\text{ ft/s}[/tex]

Distance

The distance traveled is the product of this speed and the time period.

  [tex]\dfrac{77.29\overline{3}\text{ ft}}{1\text{ s}}\times4.99\text{ s}=385.6937\overline{3}\text{ ft}\approx\boxed{385.7\text{ ft}}[/tex]

Which of the following is a proper way to enter an answer of 1.666 on the SAT SPR answer grid?
A

1/66
B

1.67
C

1.66
D

1.666

Answers

depending on if they want you to round up or not it can be 1.67. but its c or d.

The correct answer is Option B) 1.67 after rounding off.

What is rounding off a decimal number?

Rounding is a process to estimate a particular number in a context. To round a number look at the next digit in the right place, if the digit is less than 5, round down and if the digit is 5 or more than 5, round up. Rounding decimals refer to the rounding of decimal numbers to a certain degree of accuracy.

Given here, the number with decimal 1.666 , thus rounding off we get the answer a 1.67

Hence, the correct answer is Option B)

Learn more about rounding off a number here:

https://brainly.com/question/28673593

#SPJ2

A company has made a rubber ball for $0.02 per square foot. the company wants to spend a maximum of $1 each on a new ball. what is the diameter of the new ball to the nearest tenth of a foot?

Answers

The diameter of a new ball is 4.0ft

According to the statement

we have given that the the company wants to spend a maximum of $1 each.

And we have to find a diameter of a new ball.

Remember that for a sphere of diameter D, the surface area is

A = 4*pi*(D/2)^2

In this case, the cost is $0.02 per square foot, and the company wants to expend (at maximum) $1 per ball, so first we need to solve:

$0.02*A = $1

A = $1/$0.02 = 50

So the surface of the ball must be 50 square feet.

Then we solve:

50ft^2 = 4*3.14*(D/2)^2

D = 2*√(50 ft^2/(4*3.14)) = 4.0 ft

here the diameter of a ball is 4.0ft.

So, The diameter of a new ball is 4.0ft

Learn more about Diameter here https://brainly.com/question/10907234

#SPJ4

A snail crawls at the rate of 1cm in 3 minutes.how long will it take to cover 1m distance? answer asap

Answers

Answer:

Step-by-step explanation:

velocity = distance divided by time

velocity of the snail = 1cm / 3min

v of snail = 0.333 cm per minute

to calculate the time of 1 meter displacement, we use the formula t = d/V

time = distance divided by velocity

time = 1meter / 0.333cm/min

time = 100cm / 0.333cm/min

time = 300 minutes

hope it helped!

if m<0 and b>0,the graph of y=max+b does not pass through which quadrant

Answers

Answer:are u sure its not y=mx+b? And it doesn’t pass thru 1 im pretty sure

Step-by-step explanation:

Answer: Quadrant III

Explanation:

Let's pick example values for m and b

I'll go for m = -2 and b = 5. Anything works as long as m is negative and b is positive.

We go from y = mx+b to y = -2x+5

Plot this graph as shown below. The line passes through quadrants I, II, and IV. Only quadrant III is not touched at all.

Select the correct answer. Identify the expression equivalent to 4(x + x + 7) - 2x + 8 - 4 by substituting x= 1 and x = 2.

A. 6x + 11
B. 3(x + 7)
C. 2(3x + 16)
D. 3x + 16​

Answers

Answer:

C is correct

Step-by-step explanation:

I'm not sure what the substituting part has to do anything, but here is how I got the answer

4(x+x+7)-2x+8-4

4(2x+7)-2x+4

8x+28-2x+4

6x+32

Factor and you get

2(3x+16), which is answer choice C

You can check that the answer is right by plugging in x=1 and x=2 into the original equation and the answer choice, which may be what that is asking.

find the volume of a sphere with a raduis of 3 in.

Answers

Answer:

V≈113.1in³.

brainlist me

Answer is 113 cubic inches

Step by step

We know the formula is
V = 4/3 pi radius ^3
I am using 3.14 for pi

Substitute (3 in.) into radius

V = 4/3 pi 3^3
V = 4/3 pi 27
V = (4/3) (3.14) (27)
Multiply and you get

V = 113.04 cu.inches
Round to 113 cu.inches

Problem solved!
Other Questions
The hospital in patient admission rate for employees of the ramsey An effective speaker will quickly establish a reason for audience members to listen. this part of the introduction is referred to as:______ Read this character description from "the gift of the magi." della finished her cry and attended to her cheeks with the powder rag. she stood by the window and looked out dully at a gray cat walking a gray fence in a gray backyard. tomorrow would be christmas day, and she had only $1.87 with which to buy jim a present. what inference can be made about della from this description? she is upset about her options. she is concerned about the stray cat. she enjoys decorating for the holidays. she does not like being married. Compare the monthly payments and total loan costs for the following pairs of loan options. Assume that both loans are fixed rate and have the same closing costs. You need a $200,000 loan Option 1 a 30-year loan at an APR of 10% Option 2 a 15-year loan at an APR of 9.5%. Find the monthly payment for each option. The monthly payment for option 1 is S The monthly payment for option 2 is S (Do not round until the final answer Then round to the nearest cent as needed) Find the total amount paid for each option The total payment for option 1 is S The total payment for option 2 is S (Use the answers from the previous step to find this answer Round to the nearest cent as needed) Compare the two options Which appears to be the better option? OA Option 2 will always be the better option B. Option 1 is the better option, but only if the borrower plans to stay in the same home for the entire term of the loan OC. Option 1 will always be the better option OD. Option 2 is the better option, but only if the borrower can afford the higher monthly payments over the entire term of the loan 0 5 Jack works as a part-time waiter. He earns $60 per weekday and $98 per day on weekends. (a) On a particular week, he works from Tuesday to Sunday. How much was he paid for that week? (b) Find his average wage rate per day. ________ is the public relations function which involves reacting to negative events. Which events listed below would terminate a brokerage relationship where a broker represented a seller Nathan has an overinflated and unjustified sense of self-importance. He is preoccupied with fantasies of success and believes that he is entitled to special treatment from others. Nathan appears to suffer from ________ personality disorder. Group of answer choices Which of the following algorithms are used in asymmetric encryption There is a pair of parallel sides in the following shape. What is the area of the shape Describe a relationship that can be modeled by the function represented by the graph, and explain how the function models the relationship. Identify and interpret the key features of the function in the context of the situation you described in part A. choose all of the correct answers. the mechanisms through which evolution takes place are related to a set of processes that include: fossils mutation natural selection adaptation generations extinction 1.1.2 Study: Introduction to Modern World History On the world map below, identify the major regions discussed in this activity. Then provide a title that describes the significance of these regions. A ball is thrown from an initial height of 5 feet with an initial upward velocity of 31 ft/s. The ball's height (in feet) after t seconds is given by the following. h=5+31t-16t^2Find all values of t for which the ball's height is 19 feet. t= _ secondsRound your answer(s) to the nearest hundredth.(If there is more than one answer, use the "or" button.) hich of the following predictions is most likely to happen, based on the themes that have emerged sin the story?Edna will accept her role and live up to the expectations of her husband and society.Edna will further distance herself from her husband and friends as her independence grows.Mr. Pontellier will encourage his wife to develop her own interests and be more independent.Robert will return home and ask Edna to leave her husband.ONE which relation is a function Conventional genetic analyses of rrna sequences is not possible for prions because:_______ Complete the punnet square.If we were to cross a heterozygous mouse with a homozygous recessive mouse, what would the probability of offspring genotypes be ? Fill in the PERCENTAGE of the probability for EACH GENOTYPE on your show your work worksheet . Ian is borrowing $1000 from his parents to buy a notebook computer. he plans to pay them back at the rate of $60 per month. ken is borrowing $600 from his parents to purchase a snowboard. he plans to pay his parents back at the rate of $20 per month. write a system equations that can be used to determine after how many months the boys will owe the same amount. A sample of a radioactive isotope had an initial mass of 440 mg in the year 1990 and decays exponentially over time. A measurement in the year 1998 found that the sample's mass had decayed to 40 mg. What would be the expected mass of the sample in the year 2001, to the nearest whole number?